Best approach to these questions?
I understand why A is correct, but I initially chose B because I didn't notice the slight distinc...
Courtney-McClelland on June 21 at 01:23AM
  • June 2018 LSAT
  • SEC3
  • Q19
1
Reply
Why was the answer not B?
I understand why E works, it was actually my instinctive choice. However, if the staff develops i...
yassmenaraim on June 21 at 01:20AM
  • June 2018 LSAT
  • SEC3
  • Q10
1
Reply
Answer choice B
Why is B wrong if it is negated that they do not participate in voluntary census. It will destroy...
amarachicynthia on June 21 at 01:17AM
  • June 2018 LSAT
  • SEC3
  • Q7
1
Reply
Why not B
It does seem that the stimulus is assuming that Urban means Major city. Usually, Cities are descr...
Nativeguy on May 26 at 06:55PM
  • June 2018 LSAT
  • SEC3
  • Q19
1
Reply
Flaw?
Is this an equivocation flaw?
Abigail-Okereke on December 12, 2022
  • June 2018 LSAT
  • SEC3
  • Q5
1
Reply
I was in between the right and wrong answer...
I was in between A and B but I chose B which was incorrect. However, if there are a few highly re...
wbutler91 on October 5, 2022
  • June 2018 LSAT
  • SEC3
  • Q4
1
Reply
I was in between C and D...
Why is the correct answer D and not C. I predicted that the loon birds are lazy and that is why t...
wbutler91 on October 5, 2022
  • June 2018 LSAT
  • SEC3
  • Q2
2
Replies
Can you explain the difference between answers ...
When I was reading the answers, question A with the quote "without first understanding the media ...
alexdoming on August 28, 2022
  • June 2018 LSAT
  • SEC3
  • Q25
1
Reply
Further clarification Please of A
Hi, Here's why I eliminated A: Umit states: "Battery-powered vehicles have very short rang...
Mazen on July 9, 2022
  • June 2018 LSAT
  • SEC3
  • Q19
5
Replies
Explanation for (D)
I got this question wrong and didn't find the explanation of the other tutor's explanation helpfu...
christophergogo on June 10, 2022
  • June 2018 LSAT
  • SEC3
  • Q21
1
Reply
Why D and not B?
I had interpreted their disagreement was best described by B. I don't understand how D is correct...
namyasandal on July 11, 2021
  • June 2018 LSAT
  • SEC3
  • Q15
3
Replies
Could someone please explain this?
Could someone please explain this? Thanks
jingjingxiao11111@gmail.com on October 6, 2020
  • June 2018 LSAT
  • SEC3
  • Q4
3
Replies
Can someone please explain this question and an...
I did end up choosing E and I see why E works, but this question overall confused me a bit. Could...
Veda-Bhadharla on July 19, 2020
  • June 2018 LSAT
  • SEC3
  • Q5
2
Replies
Why is C wrong?
Thanks
Veda-Bhadharla on July 8, 2020
  • June 2018 LSAT
  • SEC3
  • Q21
1
Reply
Why is D wrong?
The author criticizes Rohmer, saying that her article has a polarizing effect on national politic...
Veda-Bhadharla on July 8, 2020
  • June 2018 LSAT
  • SEC3
  • Q22
1
Reply
Right answer explanation
Can someone please explain the right answer?
zia305 on June 19, 2020
  • June 2018 LSAT
  • SEC3
  • Q12
1
Reply
Why is E wrong?
Could you explain why E is wrong? Thank you!
Katie-Pleiss on June 15, 2020
  • June 2018 LSAT
  • SEC3
  • Q18
1
Reply
Why is A wrong? Thanks
Why is A wrong? Thanks
jingjingxiao11111@gmail.com on June 5, 2020
  • June 2018 LSAT
  • SEC3
  • Q18
1
Reply
EXPLANATION
I NEED HELP WITH THESE PLEASE.
toyalli on February 11, 2020
  • June 2018 LSAT
  • SEC3
  • Q13
1
Reply
A
A says major cities will not be polluted, what about smaller cities? That is what turned me off t...
tomgbean on January 16, 2020
  • June 2018 LSAT
  • SEC3
  • Q19
1
Reply